Adrianna has a court to play basketball with her friends.
The it is 600 square feet. It is 30 feet long. How many feet across is
court?​

Answers

Answer 1

Answer:

Hey there!

The area of a rectangle is the length times width.

Thus, we can write the equation, 600=30w.

Solving for the width, we get that the width is equal to 20 ft.

Let me know if this helps :)

Answer 2

Answer:

20 feet across.

Step-by-step explanation:

You will have to do a simple equation solve.

x is how many feet across the court is.

* could be our multiplying sign

600 = 30*x

Now divide 30 on both sides. 30 will cross out (since 30/30 is 1 and anything times 1 is the same number as it was before) on the right side and 600/30 is 20 so we change the 600 to 20.

That leaves 20 = x.

So it is 20 feet across.


Related Questions

When all possible differences between pairs of population means are evaluated not with an F test, but with a series of regular t tests, the probability of at least one:

Answers

Answer:

When all possible differences between pairs of population means are evaluated not with an F test, but with a series of regular t tests, the probability of at least one:

A. type I error is larger than the specified level of significance.

B. type II error is larger than the specified level of significance.

C. type I error is smaller than the specified level of significance.

D. type II error is smaller than the specified level of significance.

Answer :  Type I error is larger than the specified level of significance.( A )

Step-by-step explanation:

An F test is a test that is used to test whether the variances between pairs of populations are equal while a T test is a test used to check if a pair of population are equal not considering the fact that the variances of the population are different .

When a T test is used to evaluate all possible differences between pairs of population instead of F test there is a probability of atleast one type 1 error larger than the specified level of significance.

(x+y)^2 where x= 4 and y= -3​

Answers

1 is the answer cccccc
(4+-3)^2
(4-3)^2
1*1
answer is 1

7th grade math!!! Please help!! will mark brainlist ​

Answers

The answer is the first choice. If every 2 cm. is 4 cm., that means that the dimensions' values are doubled. Then, you can just double the dimensions. Preferably, in my opinion, this is the easier way to go. By your preference, though, you can do it the longer way as well.

There are 4 2s in the number 8.

There are 3 2s in the number 6.

If each of the 2s are 4s,

So, therefore,

Transforming the 2s to 4s,

4x4=16 cm.

3x4=12 cm.

So, this means that the final answer is the first choice. I hoped that this helped  answer your question. Enjoy your day, and take care!

Answer:

`Length=16 feet, width=12 feet

Image result for how to  use scale model step by step'

Divide the real life dimension of either length or width by that of the model. So the real life object had a length of 32m, and the model had a length of 8, then do 8(4). This is equal to 32, then divide by 2.

Hope this helps love <3

Find the length of GV¯¯¯¯¯¯¯¯ A. 43.92 B. 33.1 C. 41.45 D. 68.87

Answers

Answer:

The answer is option A

Step-by-step explanation:

Since the figure above is a right angled triangle we can use trigonometric ratios to find GV

To find GV we use cosine

cos∅ = adjacent / hypotenuse

From the question

GV is the adjacent

GC is the hypotenuse

So we have

[tex] \cos(37) = \frac{GV}{GC} [/tex]

GC = 55°

GV[tex] \cos(37) = \frac{GV}{55} [/tex]

GV = 55 cos 37

GV = 43.92495

We have the final answer as

GV = 43.92

Hope this helps you

Evaluate cosA/2 given cosA=-1/3 and tanA >0

Answers

Answer:

[tex]\bold{cos\dfrac{A}{2} = -\dfrac{1}{\sqrt3}}[/tex]

Step-by-step explanation:

Given that:

[tex]cosA=-\dfrac{1}3[/tex]

and

[tex]tanA > 0[/tex]

To find:

[tex]cos\dfrac{A}{2} = ?[/tex]

Solution:

First of all,we have cos value as negative and tan value as positive.

It is possible in the 3rd quadrant only.

[tex]\dfrac{A}{2}[/tex] will lie in the 2nd quadrant so [tex]cos\dfrac{A}{2}[/tex] will be negative again.

Because Cosine is positive in 1st and 4th quadrant.

Formula:

[tex]cos2\theta =2cos^2(\theta) - 1[/tex]

Here [tex]\theta = \frac{A}{2}[/tex]

[tex]cosA =2cos^2(\dfrac{A}{2}) - 1\\\Rightarrow 2cos^2(\dfrac{A}{2}) =cosA+1\\\Rightarrow 2cos^2(\dfrac{A}{2}) =-\dfrac{1}3+1\\\Rightarrow 2cos^2(\dfrac{A}{2}) =\dfrac{2}3\\\Rightarrow cos(\dfrac{A}{2}) = \pm \dfrac{1}{\sqrt3}[/tex]

But as we have discussed, [tex]cos\dfrac{A}{2}[/tex] will be negative.

So, answer is:

[tex]\bold{cos\dfrac{A}{2} = -\dfrac{1}{\sqrt3}}[/tex]

A city is holding a referendum on increasing property taxes to pay for a new high school. In a survey of 434 likely voters, 202 said that they would vote "yes" on the referendum. Create a 95% confidence interval for the proportion of likely voters who would vote "yes" on the referendum. Use a TI-83, TI-83 plus, or TI-84 calculator, rounding your answers to three decimal places.

Answers

Answer: 0.418 < p < 0.512

Step-by-step explanation: A 95% conifdence interval for a population proportion is given by:

[tex]p + z\sqrt{\frac{p(1-p)}{n} }[/tex]

where:

p is the proportion

z is score in z-table

n is sample size

The proportion for people who said "yes" is

[tex]p=\frac{202}{434}[/tex] = 0.465

For a 95% confidence interval, z = 1.96.

Calculating

[tex]0.465 + 1.96*\sqrt{\frac{0.465(0.535)}{434} }[/tex]

[tex]0.465 + 1.96*\sqrt{0.00057}[/tex]

0.465 ± 1.96*0.024

0.465 ± 0.047

Interval is between:

0.465 - 0.047 = 0.418

0.465 + 0.047 = 0.512

The interval with 95% of confidence is between 0.418 and 0.512.

Let f(x)=x^3-6x+3 i. find the domain of the function and f’(x) in the domain.​

Answers

Domain of a any cubic function [tex]f(x)=ax^3+bx^2+cx+d[/tex] is defined to be always [tex]\mathbb{R}[/tex].

The derivative with respect to x of your cubic function is,

[tex]\dfrac{d}{dx}f(x)=f'(x)[/tex]

to find the derivative of a polynomial function, simply take a derivative of each factor and sum them up,

[tex]\dfrac{d}{dx}x^3=3x^2[/tex] by the rule [tex]\dfrac{d}{dx}x^m=mx^{m-1}[/tex] where [tex]m\in\mathbb{R}[/tex]

[tex]\dfrac{d}{dx}-6x=-6[/tex]

[tex]\dfrac{d}{dx}3=0[/tex]

So the derivative is,

[tex]f'(x)=3x^2-6[/tex]

both derivative and the original function have equal domain.

Hope this helps :)

Step-by-step explanation:

[tex]thank \: you[/tex]

At the end of March, Jerry bought a plant that was $16.2 inches tall. During April, the plant grew 2] inches,
and during May, the plant grew another 2 inches. How tall was the plant at the beginning of June?
O 18.2 inches
O 19 inches
0 21 inches
21 inches

Answers

Answer:

20.2 inches

Step-by-step explanation:

The plant starts at 16.2 inches

The first month it grows 2 inches

16.2+2 =18.2 inches

Then the next month it grows 2 inches

18.2+2 = 20.2 inches

Sharon tried to solve an equation step by step.

9
9
15
5


=−3(e−2)
=−3e+6
=3e
=e


Step 1
Step 2
Step 3


Find Sharon's mistake.
Choose 1 answer:
Choose 1 answer:

Answers

Answer:

step 2

Step-by-step explanation:

9 = -3(e - 2)

9 = -3e + 6

9-6 = -3e

3 = -3e

divide both sides by -3

-1 = e

find the circle through (-4,sqrt(5) with center (0,0)

Answers

Answer:

Circle Equation : x² + y² = 21

Step-by-step explanation:

So we know that this circle goes through the point ( - 4, √5 ), with a center being the origin. Therefore, this makes the circle equation a bit simpler.

The first step in determining the circle equation is the length of the radius. Applying the distance formula, the radius would be the length between the given points. Another approach would be creating a right triangle such that the radius is the hypotenuse. Knowing the length of the legs as √5 and 4, we can calculate the radius,

( √5 )² + ( 4 )² = r²,

5 + 16 = r²,

r = √21

In general, a circle equation is represented by the formula ( x - a )² + ( y - b )² = r², with radius r centered at point ( a, b ). Therefore our circle equation will be represented by the following -

( x - 0 )² + ( y - 0 )² = (√21 )²

Circle Equation : x² + y² = 21

Estimate the mean exam score for the 50 students in Prof. Burke's class.

Score

f

40 but less than 50

21



50 but less than 60

39



60 but less than 70

40



70 but less than 80

34



80 but less than 90

28



Total

162



Group of answer choices

65.56

63.78

64.89

62.34

Answers

The mean of the exam score is 65.56

The given exam data is as follows;

Possible Score range ----------- frequency (f) -------- score (x) -----------(fx)

40 - 50   -------------------------  21 ------------------- -----45 -------------- 945

50 - 60  --------------------------- 39------------------------55 ---------------2145

60 - 70 ----------------------------- 40---------------------- 65 ---------------- 2600

70 - 80 ------------------------------ 34 --------------------- 75 ----------------- 2550

80 - 90 ------------------------------ 28 --------------------- 85 ---------------- 2380

Note:[tex]score (x) = \frac{sum \ of \ the \ range }{2} , \ example \ \frac{40+49.99}{2} \approx 45[/tex]

The sum of the frequency (f) = 162

The sum of fx, ∑fx = 10620

The mean of the exam score:

[tex]\bar x = \frac{\Sigma fx }{\Sigma f} = \frac{10620}{162} = 65.56[/tex]

                                               

Therefore, the mean of the exam score is 65.56

To learn more about grouped mean calculation please visit: https://brainly.in/question/20735794

What value does the 2 in the number 0.826?

Answers

Answer:

.02

Step-by-step explanation:

2 is in "Hundredths' place in .826

So, the number is multiplied with 1/100 or .01

=> 2 x 1/100

=> 2/100

=> .02

=> 2 x .01

=> .02

The value of 2 in .826 is .02

1. In the past, Sam cashed his paycheck each month at Ready Cash, a check cashing service that
charges a 5% fee. He recently opened a checking account at Bank of America so he can now
deposit and/or cash his paycheck without a fee. If Sam is making $28,500 per year, how much will
he save by not going to Ready Cash anymore?

Answers

Step-by-step explanation:

28000 ÷ 100

=280

280 × 5

=1400

13
R
S
12
What's the length of QR?
A) 1
B) 17.7
C) 6.7
OD) 5

Answers

Answer:

5

Step-by-step explanation:

This is a right triangle, so we can use the Pythagorean theorem

a^2+b^2 = c^2

where a and b are the legs and c is the hypotenuse

QR^2 + 12^2 = 13^2

QR^2 +144 =169

QR^2 = 169-144

QR^2 =25

Take the square root of each side

QR = sqrt(25)

QR =5

Solve the right triangle.
a = 3.3 cm, b = 1.7 cm, C = 90°
Round values to one decimal place.

Answers

Answer:

A = 62.7°B = 27.3°c = 3.7

Step-by-step explanation:

  tan(A) = a/b = 3.3/1.7

  A = arctan(33/17) ≈ 62.7°

  B = 90° -A = 27.3°

  c = √(a²+b²) = √(3.3² +1.7²) = √13.78

  c ≈ 3.7

What's 955 ÷ 35 ?
[tex]955 \div 35[/tex]

Answers

Answer:

Hello

Step-by-step explanation:

955÷35

27.2857142857

Or

35/955

35)955(27

-945

________

10

I hope this works♥️

The given line segment has a midpoint at (−1, −2). On a coordinate plane, a line goes through (negative 5, negative 3), (negative 1, negative 2), and (3, negative 1). What is the equation, in slope-intercept form, of the perpendicular bisector of the given line segment? y = −4x − 4 y = −4x − 6 y = One-fourthx – 4 y = One-fourthx – 6

Answers

Answer:

y = -4x - 6.

Step-by-step explanation:

We are given (-5, -3), (-1, -2), and (3, -1) for points of a line. First, we need to find the slope.

(-2 - -3) / (-1 - -5) = (-2 + 3) / (-1 + 5) = 1 / 4.

A perpendicular bisector would have a slope of -4, which is the negative reciprocal of 1/4.

Now that we have the slope, we can say that the equation is y = -4x + b. To find what is b, we can say that y = -2 and x = -1.

-2 = -4(-1) + b

-2 = 4 + b

b + 4 = -2

b = -6

So, the equation of the perpendicular bisector is y = -4x - 6.

Hope this helps!

Answer:

y = -4x - 6.

Step-by-step explanation:

Just took the test and got it right

PLEASE HELP ASAP, I WILL GIVE BRAINLIEST!

9. What is another name for BD?
10. What is another name for AC?
11. What is another name for ray AÉ?
12. Name all rays with endpoint E.
13. Name two pairs of opposite rays.
14. Name one pair of rays that are not opposite rays.

Answers

Answer:

#9. Segment DB

#10 Segment CA

#11. Ray EA

#12. Ray EB, Ray EC, Ray ED, Ray EA  

#13 Ray EA & Ray EC

#14. Ray EB and Ray EC

Step-by-step explanation:

For future reference, if you are asked to give another name for a segment, just flip the letters around (same for rays.).

Opposite Rays are rays that share 1 common endpoint, but extend in opposite directions ( together they make a 180 degree angle). Ray EB & EC share an endpoint, but they do not extend in opposite directions.

Hope this helps!  

Rhombus J K L M is shown. The length of J K is 2 x + 4 and the length of J M is 3 x. What is the length of a side of rhombus JKLM? 4 units 8 units 12 units 16 units

Answers

Answer:

12 units

Step-by-step explanation:

Since all of the sides of a rhombus are congruent, JK = JM which means:

2x + 4 = 3x

-x = -4

x = 4 so 3x = 3 * 4 = 12

What is the percentage of 204 over 1015, 1 over 8120, 1 over 5832, and 1 over 6?

Answers

Answer:

204/1015 (irreducible) = 20.1%

1/8120 (irreducible) = 0.01232%

1/5832 (irreducible) = 0.01715%

1/6 (irreducible) = 16.67%

Step-by-step explanation:

Start with the number 2380.
Divide by 10,
The 8 will end up in the _____ place.

Answers

Answer:

The 8 will end up in the ones place.

Step-by-step explanation:

If you want to divide 2380 by 10… sry gtg

The 8 will end up in the "ones place".

How can we interpret the division?

When 'a' is divided by 'b', then the result we get from the division is the part of 'a' that each one of 'b' items will get. Division can be interpreted as equally dividing the number that is being divided into total x parts, where x is the number of parts the given number is divided.

We need to find the 8 will end up in which place

A negative divided by a negative is positive, then;

2380/ 10 = 238

Therefore, The 8 will end up in the _ ones_ place.

Learn more about division here:

brainly.com/question/26411682

#SPJ2

A system of equations consists of the two equations shown.
{4x+5y=18
6x−5y=20
Which procedure will produce a single equation in one variable? Select all the procedures that apply.
A. Subtract the first equation from the second equation.
B. Subtract the second equation from the first equation.
C. Multiply the first equation by 18; multiply the second equation by 18; add the equations.
D. Multiply the first equation by − 6; multiply the second equation by 4; add the two equations.
E. Multiply the first equation by 3; multiply the second equation by − 2; add the two equations.
F. Multiply the first equation by 3; multiply the second equation by 2; subtract the equations in any order.

Answers

Answer:

C, D, E and F

Step-by-step explanation:

Given

4x+5y=18

6x−5y=20

Required

Determine which procedure will result in a single equation in one variable

To do this; we'll test each of the options

A. Subtract the first equation from the second equation.

[tex](6x - 5y=20) - (4x+5y=18)[/tex]

[tex]6x - 4x - 5y - 5y = 20 - 18[/tex]

[tex]2x - 10y = 2[/tex] --- This didn't produce the desired result

B.  Subtract the second equation from the first equation.

[tex](4x+5y=18) - (6x - 5y=20)[/tex]

[tex]4x - 6x + 5y + 5y =18 - 20[/tex]

[tex]-2x + 10y = -2[/tex] --- This didn't produce the desired result

C. Multiply the first equation by 18; multiply the second equation by 18; add the equations.

First Equation

[tex]18 * (4x+5y=18)[/tex]

[tex]72x + 90y = 324[/tex]

Second Equation

[tex]18 * (6x - 5y=20)[/tex]

[tex]108x - 90y = 360[/tex]

Add Resulting Equations

[tex](72x + 90y = 324) + (108x - 90y = 360)[/tex]

[tex]72x + 108x + 90y - 90y = 324 + 360[/tex]

[tex]72x + 108x = 324 + 360[/tex]

[tex]180x = 684[/tex] --- This procedure is valid

D. Multiply the first equation by − 6; multiply the second equation by 4; add the two equations.

First Equation

[tex]-6 * (4x+5y=18)[/tex]

[tex]-24x - 30y = -108[/tex]

Second Equation

[tex]4 * (6x - 5y=20)[/tex]

[tex]24x - 20y = 80[/tex]

Add Resulting Equations

[tex](-24x - 30y = -108) + (24x - 20y = 80)[/tex]

[tex]-24x + 24x - 30y -20y = -108+ 80[/tex]

[tex]-50y = -28[/tex]

[tex]50y = 28[/tex]  --- This procedure is valid

E. Multiply the first equation by 3; multiply the second equation by − 2; add the two equations.

First Equation

[tex]3 * (4x+5y=18)[/tex]

[tex]12x + 15y = 54[/tex]

Second Equation

[tex]-2 * (6x - 5y=20)[/tex]

[tex]-12x + 10y = -40[/tex]

Add Resulting Equations

[tex](12x + 15y = 54) + (-12x + 10y = -40)[/tex]

[tex]12x - 12x + 15y - 10y =54 - 40[/tex]

[tex]5y = 14[/tex]  --- This procedure is valid

F. Multiply the first equation by 3; multiply the second equation by 2; subtract the equations in any order

First Equation

[tex]3 * (4x+5y=18)[/tex]

[tex]12x + 15y = 54[/tex]

Second Equation

[tex]2 * (6x - 5y=20)[/tex]

[tex]12x - 10y = 40[/tex]

Subtract equation 1 from 2 or 2 from 1 will eliminate x;

Hence, the procedure is also valid;

The work shows how to use long division to find?

Answers

Answer:

remainder = [tex]\frac{1}{x-2}[/tex]

Step-by-step explanation:

The remainder is the value of the subtraction of the last 2 lines in the procedure.

  5x - 9

- (5x - 10)

That is

5x - 5x = 0 and - 9 - (- 10) = - 9 + 10 = 1

over the divisor x- 2 , gives

remainder = [tex]\frac{1}{x-2}[/tex]

The sum of the product of a number x and 14, and 13

Answers

Answer:

ax+182

Step-by-step explanation:

a*x+14*13

ax+182

What should be added to p to get p+q?​

Answers

Answer:

Step-by-step explanation:

The correct answer is q

Because we have been given the equation p + q already so if we observe the equation carefully we see that if we add the variable q to the variable p we get the answer p + q

For the following graph, state the polar coordinate with a positive r and positive q (in radians). Explain your steps as to how you determined the coordinate (in your own words). I'm looking for answers that involve π, not degrees for your angles. State the polar coordinate with (r, -q). Explain how you found the new angle. State the polar coordinate with (-r, q). Explain how you found the new angle. State the polar coordinate with (-r, -q). Explain how you found the new angle.

Answers

Answer:

Points : ( 8, - 2π/3 ), ( - 8, π/3 ), ( - 8, - 5π/3 )

Step-by-step explanation:

For the first two cases, ( r, θ ) r would be > 0, where r is the directed distance from the pole, and theta is the directed angle from the positive x - axis.

So when r is positive, we can tell that this point is 8 units from the pole, so r is going to be 8 in either case,

( 8, 240° ) - because r is positive, theta would have to be an angle with which it's terminal side passes through this point. As you can see that would be 2 / 3rd of 90 degrees more than a 180 degree angle,or 60 + 180 = 240 degrees.

( 8, - 120° ) - now theta will be the negative side of 360 - 240, or in other words - 120

Now let's consider the second two cases, where r is < 0. Of course the point will still be 8 units from the pole. Again for r < 0 the point will lay on the ray pointing in the opposite direction of the terminal side of theta.

( - 8, 60° ) - theta will now be 2 / 3rd of 90 degrees, or 60 degrees, for - r. Respectively the remaining degrees will be negative, 360 - 60 = 300, - 300. Thus our second point for - r will be ( - 8, - 300° )

_________________________________

So we have the points ( 8, 240° ), ( 8, - 120° ), ( - 8, 60° ), and ( - 8, - 300° ). However we only want 3 cases, so we have points ( 8, - 120° ), ( - 8, 60° ), and ( - 8, - 300° ). Let's convert the degrees into radians,

Points : ( 8, - 2π/3 ), ( - 8, π/3 ), ( - 8, - 5π/3 )

For the functions f(x)=4x+5 and g(x)=6x+4, find (f∘g)(0) and (g∘f)(0).

Answers

Answer:

Step-by-step explanation:

f(g(0))=

g(0)= 6(0) + 4 = 0 + 4 = 4

f(4)= 4(4)+5 = 16 + 5 = 21

g(f(0))=

f(0)= 4(0)+5 = 0+5 = 5

g(5)= 6(5)+4 = 30+4= 34

1/3 is part of which set of numbers?

Answers

Answer:

[tex] \frac{1}{3} [/tex]Rational number as denominator is not equal to zero and numerator is a integer.

Rational numbers. denoted by [tex] \mathbb Q[/tex]

1/3 is clearly not a natural number or integer.

it is a fraction, =0.333 , it fits the definition of rational number ([tex] \frac pq [/tex]).

I am performing a before and after evaluation on 30 students who have taken a keyboarding class. I want to see if the course improved their words per minute keyed.

Required:
a. State the Null and Alternate Hypothesis.
b. The statistic that I would use is:_________
c. What would my t critical be for this calculation at a 0.10 level of significance?
d. If my t calculated = 1.62, would I reject or fail to reject the null hypothesis?

Answers

Answer:

a)

H₀ : µd = 0  

H₁ : µd < 0  

b)

The test statistic is

tₙ₋₁ = α / s√n

c)

at 0.10 level of significance,

tₙ₋₁ , ₐ

t₃₀₋₁ , ₀.₁₀ = t₂₉, ₀.₁₀ = 1.311

d)

given that  T(critical) = 1.62

∴ T(critical) = 1.62 > t₂₉, ₀.₁₀ = 1.311

at 10% level of significance,

REJECT H₀

Since 1.62 > 1.311, we can reject the null hypothesis.

After a 75% reduction, you purchase a new clothes dryer for $200. What was the original price of the clothes dryer?

Answers

Answer:

$800

Step-by-step explanation:

Let the original price be $x.

75% reduction ----- 100% -75%= 25%

25%x= 200

[tex] \frac{25}{100} x = 200 \\ x = 200 \div \frac{25}{100} \\ x = 200 \times \frac{100}{25} \\ x = 800[/tex]

Thus, the original price of the clothes dryer is $800.

Answer:

$800

Step-by-step explanation:

Let the original price be x.

Final price=100%-75%

                  =25%

x-75%=200

x=200 x 100/75

x=8 x 100

x=800

Thank you!

Other Questions
The transfer of political attitudes and beliefs __________. Group of answer choices is more likely to occur from parents to children never occurs from children to parents is exceptionally rare in families is more likely to occur from children to parents is just as likely to occur from children to parents as it is to occur from parents to children You are rolling a 6-sided number cube with the numbers 1 through 6. Which of the following represents the probability of rolling an even number?01/61/21 An operator wants to determine the standard deviation for a machine relative to its ability to produce windshield wipers conforming within their specifications. To do this, she wants to create a p-chart. Over a month's time, she tests 100 units every day and records the number of manufacturing defects. The average proportion of non-conforming windshield wipers is found to be 0.042. What is the standard deviation of this sample Both acids and bases produce ions when dissolved in water. Propose a way to describe whathydroxide ions (OH-) and H+ (hydrogen) ions are. What do they have to do with acids andbases and the pH scale? convert 0.5 seconds to milliseconds and .02 km to meters Pls help me with this Answer.... Describe Mourad in your own words ? a) How can you say that Rip Van Winkle was a good man and a generousneighbor? What is the smallest 3-digit palindrome that is divisible by both 3 and 4? How is irony used in The Innocents Abroad? Support your answer with evidence from the selection. What is the slope of the line x = 4? Three capacitors C1 = 10.7 F, C2 = 23.0 F, and C3 = 29.3 F are connected in series. To avoid breakdown of the capacitors, the maximum potential difference to which any of them can be individually charged is 125 V. Determine the maximum energy stored in the series combination. I WANT TO PASS HIGH SCHOOL PLEASE HELPGiven that the quadrilateral EFGH is a trapezoid, mH = 62 and mHEG = 64 calculate mFEG what is the equation pls no guessing brainliest includeD!!!!!!!!!! The color cyan has a frequency of 5.902x10^14 What is the wavelength in nm ? Record your answer with 2decimals. Please help explanation if possible If a falling object is covering 50 metre distance in 5 seconds, how much time it will take to cover 72 metres? Define the term gender imbalance and state two negative effects that this may have in the workplace? If the cost of labor decreases the isocost line will A. stay the same. B. shift inward in parallel fashion. C. rotate outward around the point where only capital is employed in production. D. shift outward in parallel fashion. Why is it important to wear loose-fitting clothing when exercising? What does 2x+1=3x-4 equal?